*NURSING > APEA > NR 603 Midterm APEA-Reviewed Q&A. completed FOR Grade A (All)

NR 603 Midterm APEA-Reviewed Q&A. completed FOR Grade A

Document Content and Description Below

APEA Questions 1. A patient report that she is taking kava kava regularly for anxiety with the results. What should the nurse practitioners evaluate? a. Liver function studies 2. A person with 20/... 80 vision: HAS POORER VISION THAN SOMEONE WITH 20/40 VISION 3. One exception to the recommendation to limit dietary fat intake is CHILDREN UNDER 2 YEARS OF AGE 4. The most effective treatment for anemia of chronic disease is: TREATMENT OF UNDERLYING DISEASE Rationale: The key phrase is “anemia of chronic disease” and with this, the pt’s chronic disease has caused the deficiency so once you correct and or treat the chronic disease, then the anemia will correct itself. 5. Salicylates and other non-steroidal anti-inflammatory agents have all of the following actions EXCEPT- ANTI-EMETIC EFFECTS VIA SUPPRESSION OF GASTRIC ACID SECRETION 6. Which of the following does NOT increase a woman’s risk of developing cervical cancer? NULLIPARITY 7. All of the following principles apply to drug therapy for elders except ENCOURAGE THE ELDERLY TO CONTINUE SELF-MEDICATION ADMINISTRATION TO FACILITATE INDEPENDENCE AND SELF-SUFFIENCY 8. A female patient believes that she was exposed to HIV through sexual intercourse several months ago. She asks for “the test for AIDS.” Which statement is NOT correct? THE ELISA IS USED FOR SCREENING, BUT MAY YIELD A FALSE NEGATIVE THIS SOON AFTER EXPOSURE 9. The nurse practitioner teaches the parents of a 2-month-old infant with GERD that treatment typically includes all of the following interventions except MAINTAINING THE USUAL FEEDING REGIMEN AND BURPING THE INFANT WELL WHEN THE FEEDING IS COMPLETED 10. A widowed 85-year-old was recently moved from her home of 65 years to a bedroom in her daughter’s home. According to reports by family members, she is now“cantankerous” gets “mixed up” easily and cries for no apparent reason. Factors in the client history that are consistent with delirium may include all of the following except a. INSIDIOUS ONSET AND CHRONIC PROGRESSIVE COURSE 11. The two tests that can indicate current infection with hep B are: a. Presents of hep B antigen and IgM 12. A characteristic of elders which affects the pharmacotherapeutics of drug therapy in that population is an increase in PERCENT OF BODY FAT 13. An elder’s caretaker is planning the elder’s menu. The elder is on a bland diet. The nurse practitioner reviews the meal plans and notes that which of the following is NOT appropriate? a. Coffee, tea, carbonated sodas, Orange juice 14. The leading cause of death from injury in childhood is MOTOR VEHICE ACCIDENTS 15. Therapeutic International normalized ratio (INR) for a patient taking warfarin (coumadin) for a chronic atrial fibrillation is expected to be: a. Increased 16. The most accepted recommendation regarding a skin cancer prevention is: a. Avoidance of excessive sun exposure 17. Antibiotic administration has been demonstrated to be of little benefit in the treatment of which of the following disease processes? a. Acute bronchitis 18. An anxious parent brings her three-day old female infant to see the nurse practitioner because of the serosanguineous vaginal discharge. The nurse practitioner tells the mother: a. This is a normal finding in female newborns 19. A common finding associated with temporal arteritis is: a. Severe headache20. What information would best help the parent who is concerned about a school age child eating habits? a. Establish a consistent schedule for meals and allow the child to participate in meal planning 21. Which of the following scenarios in a 75-year-old patient would be inappropriately managed in an outpatient setting (i.e. the patient should be referred for hospital admission)? a. UNSTABLE ANGINA 22. A patient with diabetes brings his glucose diary from the past seven days for the nurse practitioner to review and evaluate. What changes should be made? Day one through seven AM 67, 52, 61, 48, 39, 68, 70 PM 138, 161, 148, 168, 121, 142, 176. a. Decreased the PM dose of NPH insulin 23. Appropriate nutritional guidance for the pregnant patient is: a. Increase caloric intake buy 300 cal/day and add iron and folic acid supplementation 24. The most successful patient interview is likely to: a. Be a collaborative process 25. RhoGAM is not administered to an Rh-negative mother with negative anti-d antibodies WITHIN 72 HOURS OF BIRTH OF AN RH-NEGATIVE INFANT 26. The most common presentation of thyroid cancer is A SOLITARY THYROID NODULE 27. A benign thyroid nodule, that’s painless presents as SMOOTH 28. Initial treatment of a child presenting with a severe head injury is: a. Aimed at resuscitation, then maintenance of oxygenation and blood flow 29. Which red blood cell (RBC) index is most useful for differentiating types of anemias? a. Mean corpuscular volume (MCV) 30. A 31-year-old female is informed that her pregnancy test is positive, and she is about three weeks pregnant. The nurse practitioner is very concerned about the patient reported history of alcohol use. She correctly tells the patient that babies born to women who drink alcohol during pregnancy are at risk for all of the following except:a. Post maturity delivery 31. According to Erikson, the developmental task of the early adult is: a. Ego integrity vs. despair 32. The parents of a two-year-old Report that she is not saying any words, but make sounds, babbles, and understand simple commands from her parents. The parents are not concerned. The nurse practitioner responds: a. Your child should be saying a few words by this time. She should be referred for further assessment 33. Which group is considered to be at high risk for the development of testicular tumors? a. Late adolescence to early adulthood (you need to know the actual age groups) 34. An urgent call is received from a patient’s wife who states that her husband is having chest pain I’m relieved by three nitroglycerin tablets. The nearest hospital is 40 minutes away. Besides having the patient reach the hospital as soon as possible, what intervention can the nurse practitioners suggest might influence a positive outcome? A. Have the patient to an aspirin on the way to the hospital 35. Which of the following postpartum mothers should receive RhoGAM (BayRho-D) to prevent Rh iso-immunization? a. A G1 P1 mother who is Rh negative without Rh antibodies; cord blood is Coombs negative; newborn is Rh positive. 36. A patient with a history of alcohol abuse presents with acute nausea, vomiting, and severe retrosternal pain that radiates to the back. Based on the most probable diagnosis, the nurse practitioner would appropriately order a CBC, chemistry profile, and: a. serum amylase and lipase, and liver function studies 37. A female patient asks, “how do I calculate my ideal body weight?” The nurse practitioner response: a. Start with 100 pounds, and add 5 pounds for every inch in height over 5 feet 38. A 35-year-old woman with prior history of extreme nervousness has been treated with diazepam (valium) for four weeks. She reports improvement. Today her pulse is 112bpm and blood pressure is 130/92 mmHg. She is 5’4” in weighs 105 pounds. What is the most likely diagnosis?a. Hyperthyroidism 39. Which of the following is not a common early sign of benign prosthetic hyperplasia (BPH)? a. Increased force of urine flow 40. Acute idiopathic thrombocytopenia purpura (ITP) is most commonly seen in childhood following: a. Acute infection 41. The nurse practitioner diagnosis epididymitis in a 24-year-old sexually active male patient. The drug of choice for treatment of this patient is: a. Oral doxycycline (Vibramycin) plus intramuscular ceftriaxone 42. A 55-year-old man is diagnosed with basal cell carcinoma. The nurse practitioner correctly tells him: a. It can be cured with surgical excision or radiation therapy. 43. A characteristic of delirium that is typically absent and dementia is: a. Acute onset of confusion in a previously alert and oriented patient 44. A 42-year-old male executive has been diagnosed with depression. He is otherwise healthy. And appropriate initial treatment choice is a: a. Selective serotonin reuptake inhibitor (SSRI) 45. the CAGE/ AUDIT questionnaire is useful to assess: a. alcohol use disorders (but you need to know a little more about 46. A 16-year-old athlete complains of pain underneath his heel every time he walks. There is verrucous surface level with the skin of the heel. What pharmacologic intervention should the nurse practitioner prescribed for this patient? a. Salicylic acid plasters 47. Previously immunized elderly patients should receive a tetanus and diphtheria (Td) booster every: a. 10 years 48. The nurse practitioner is canceling a young woman who desires pregnancy. She discontinued her oral contraceptives for months ago. Her urine pregnancy test is negative. She expresses concern that she might have an infertility problem. The nursepractitioner accurately tells her that a couple is not considered infertile until there has been unprotected intercourse without conception for what period of time? A. One year 49. When a patient presents with symptoms of acute gallbladder disease, what is the appropriate nurse practitioner action? a. Order abdominal ultrasound 50. An eight-year-old female has been brought to the nurse practitioner on five occasions in the past nine weeks with a complaint of abdominal pain. The evaluation each time is negative, but her mother is convinced the child is truly experiencing pain. The most likely diagnosis is: a. Recurrent abdominal pain 51. What is the most common cause of Cushing’s syndrome? a. Administration of a glucocorticoid or ACTH 52. Which of the following maternal situations is considered an absolute contraindication to breast-feeding? a. Early HIV infection 53. A three-year-old male patient is being evaluated by a nurse practitioner for frequent respiratory infections and fatigue. His laboratory test show thrombocytopenia, granulocytopenia, and anemia. Which of the following is consistent with these lab bindings? a. Acute lymphocytic leukemia (ALL) 54. Factors known to be associated with early sexual activity include: a. Lower socioeconomic status, use of tobacco, alcohol or other drugs, and single parent household 55. A 26-year-old female presents with elbow pain that is described as aching in burning. There is point tenderness along the lateral aspect of the elbow and painful passive flexion and extension. She reports she has been playing tennis almost daily for the past month. The most likely diagnosis is: A. Lateral epicondylitis ------tennis elbow56. A two-year-old African-American child is being evaluated by the nurse practitioner. His clinical appearance and history are consistent with sickle cell anemia. What is the appropriate laboratory test to diagnose sickle cell anemia? a. Hemoglobin electrophoresis 57. Which activities are not characteristic of preschool children? a. Always follow rules during playground games 58. What type of question is most useful when opening an interview? a. Open ended 59. All of the following agents/ medication classes are generally contraindicated in pregnancy except: a. Cephalosporins, parenteral insulin, parenteral heparin. 60. A 35-year-old female with a history of mitral valve prolapse is scheduled for routine dental cleaning. According to the 2007 American Heart Association’s guidelines for endocarditis prophylaxis, what would you advise this patient? a. She does not need prophylaxis for any dental procedure 61. What diabetic complications result from hyperglycemia? a. (1) Retinopathy, (3) peripheral neuropathy, (4) accelerated atherogenesis 62. Which of the following diagnostic assessments must be considered before developing any plan of care for a 27-year-old female presenting to a family practice clinic? a. Pregnancy test 63. The process by which a professional association confers recognition that a licensed professional has demonstrated mastery of a specialized body of knowledge and skills is termed: a. Certification 64. A 15-year-old male has a history of cryptorchidism which was surgically repaired. Because of this information, it is essential for a nurse practitioner to teach him about: a. Testicular self-examination65. A 30-year-old female comes into a clinic with classic signs and symptoms of appendicitis. The nurse practitioner fails to refer the patient tree surgeon. D appendix ruptured and the woman dies. This is an example of: a. Malpractice 66. And elderly patient is being seen in the clinic for a complaint of “weak spells” relieved by sitting or lying down. The nurse practitioner checks the patient’s blood pressure while he is lying, sitting, and standing. The nurse practitioner suspects: a. Orthostatic hypertension 67. The nurse practitioner knows that the frail elderly alright high risk for malnutrition. Indicators of malnutrition in this population included all of the following except: a. Hunger and increased appetite 68. The patient reports, “something flew in my eyes” about an hour ago while he was splitting logs. If there was a foreign body in his eye, the nurse practitioner would expect to find all except: a. A purulent discharge 69. Inappropriate initial treatment for external genital warts caused by human papilloma virus (HPV) in a non-pregnant patient is: a. Topical trichloroacetic acid (TCA) 70. A new patient presents to the nurse practitioner clinic stating she wants a second opinion. She started propylthioutacil (PTU, Propyl-Thyracil) 75 mg three times today a week ago as therapy for newly diagnosed Graves’ disease. She still feels irritable and jittery. How should the nurse practitioner respond? a. Inform the patient that improvement requires 2 to 3 weeks 71. What advice should be given to the parents of an infant at risk for SIDS? a. Place the baby in the supine position for sleeping 72. A six-month-old patient with type III metatarsus adductus (i.e., rigid and does not correct to neutral) should be treated with: a. Surgical intervention 73. Patient should be advised to purchase the same brand name for each of the following drugs except:a. Hydrochlorothiazide 74. Which activity during the first trimester could be deleterious to the fetus? a. Dieting 75. The mother of an 18-month-old child is in the office. She is not yet immunized her child because the child’s grandmother “doesn’t believe in that.” The nurse practitioner should: a. Provide education and recommend beginning of the series of vaccines 76. The nurse practitioner is conducting a routine health assessment of a 28- year-old patient. He states he is very “Health conscious” and that he ran 6 miles daily. He denies current Health problems and his past medical history is unremarkable. Routine urinalysis reveals proteinuria. What is the most likely cause of the proteinuria? a. Exercise 77. What is the earliest age that MMR immunization can be administered? a. One year 78. The nurse practitioner examines a six-year-old who has had a sore throat and fever for less than 24 hours. Based on the most common cause of pharyngitis in this age group, the most appropriate action is to: a. Encourage supportive and symptomatic care 79. Physical examination findings consistent with emphysema include all of the following except: a. Pallor and cyanosis of the mucosa nail beds 80. Which choice below is not a contraindication to receiving the diphtheria, pertussis, and tetanus (DPT) vaccine? a. Current antibiotic use 81. All of the following are included in the differential diagnosis of hyperthyroidism except: a. Plummer’s disease 82. A middle-aged patient has a history of smoking, chronic cough, and sputum production. He reports chronic dyspnea. A diagnosis of COPD should be confirmed with: a. Spirometry83. A patient requests information about foods to include in his diet while vacationing in Mexico to avoid “Montezuma’s revenge.” The nurse practitioner should advise him to ingest only: a. Cooked vegetables in soup, bottled water, and wine 84. An eight-month-old male presents with hemarthrosis of both knees and hematuria. The parents gave no history of trauma but say “he has always bruised easily.” The most likely diagnosis is: a. A type of hemophilia 85. Of the following for malpractice issues for which an APRN may incur liability, which is the most prevalent cause of action? a. Failure to refer when the APRN and skills are exceeded 86. Which of the following factors is (are) associated with increased incidence of cervical cancer? a. Smoking and sexual activity at an early age with multiple partners 87. Following the finding of prostate gland abnormalities on DRE, the nurse practitioner orders appropriate labs. When preparing to view lab reports with the patient, and Nurse practitioner knows all of the following are true except: a. Normal PSA it’s 10ng/ml or less 88. Which characteristic is not indicative of heart failure? a. Purulent sputum 89. A 32-year-old male patient complains of urinary frequency and burning on urination for three days. Urinalysis reveals bacteriuria and positive nitrates. He denies any past history of urinary tract infections. The initial treatment should be: a. Trimethoprim-sulfamethoxazole (Bactrim, sulfatrim) for 7 to 10 days 90. On trans illumination of the scrotum, the nurse practitioner identifies a hydrocele in a three-month-old infant. What course of action should be taken? a. Monitor the infants until 12 months of age 91. Which of the following is an appropriate drug for initial treatment of gastroesophageal reflux disease (GERD) in adults? a. Famotidine (Pepcid)/ You need to know the drug class92. The major health risk associated with oral contraceptive therapy is? a. Cardiovascular events 93. The most effective treatment of primary and secondary encopresis focuses on: a. Establishment of a regular bowel routine 94. A 55-year-old male patient complains of blood in his urine for the past two weeks. He denies dysuria. Urinalysis is positive for red blood cells and negative for leukocytes and nitrates. The initial evaluation of this patient should include: a. Renal ultrasonography 95. A 50-year-old non-smoker with no comorbidities presents to the clinic an is diagnosed with community acquired pneumonia. His vital signs are normal except for temperature 101.6°F. a sputum specimen is collected and sent for culture and sensitivity. What action should the nurse practitioner take today? a. Start Clarithromycin (Biaxin) 500 mg BID for five days 96. ??????? a. Influenza vaccine should be given annually to all persons 65 years or older, antipersons six months or older who are And chronic care facilities, or who suffer chronic cardiopulmonary disorders, metabolic disorders, hemoglobinopathies, or immunosuppression. 97. Simple cases of impetigo and folliculitis are usually successfully treated with: a. Topical antibiotics 98. A patient on oral contraceptives reports that she has forgotten to take her pill the last two days in A row. What should you advise her? 1-take two tablets daily for the next two days, then resumed a regular schedule. 2-begin a new compact of tablets starting the first day of the next cycle. 3-use an additional method of contraception for the rest of the pill pack.4-take one tablet, discard the second, and continue as scheduled. a. 1 and 3 99. A three-year-old has enlarged, warm, tender cervical lymph nodes, indicating: a. Infection proximal to the nodes100. A 62-year-old female has been diagnosed with osteoporosis. She refuses hormone replacement therapy as well as new medication shown to increase bone density. Important education for this patient should include: a. The benefits of increasing calcium daily and daily weight bearing exercises 101. The tricyclic antidepressants (TCAs) and selective serotonin reuptake inhibitors (SSRIs) have demonstrated efficacy in the treatment of each of the following conditions except: a. Gastroesophageal reflux 102. Which finding below would be unusual and patient with diabetic retinopathy? a. Papilledema 103. A two-year-old child is diagnosed with hemophilus influenza type B. The child’s parents will not allow any of their children to be immunized. There are two other children in the home, ages one year and three years, how should the nurse practitioner manage contacts? a. Rifampin prophylaxis should be given to all household contacts, including the adults. 104. Atenolol (Tenormin) should be avoided in: a. A 43-year-old female with asthma 105. The results of the Weber tuning fork test is the lateralization of sound to the right ear. This finding indicates: a. Conduction problem in the right here 106. A male patient with chronic atrial fibrillation takes a generic brand of Coumadin (warfarin). He should report all of these to his healthcare provider except: a. One missed dose of warfarin 107. The side effect of the medroxyprogesterone (depo-Provera) that often leads women to discontinue use is: a. Weight gain 108. The mother of a two-week-old infant with hypospadias request circumcision for her infant. The nurse practitioner’s best response to this mother should be:a. Explain why an infant with hypospadias should not be circumcised 109. A patient complains of “stomach pains” on and off for the past month. In distinguishing between a gastric and duodenal ulcer, what question is least important to ask? a. Have you been out of the country in the past several months 110. Enlargement of the scrotum and testes with little change in size of the penis characterizes which tanner stage of sexual development? a. Tanner 2 111. Which of the following ecchymotic lesions in a four-year-old patient most likely represents a possible bleeding disorder: a. 7 cm on the abdomen 112. The mother of a six-month-old infant asked about the use of an infant walker. The most appropriate response is to: a. Discourage the use of walkers and encourage parental holding and floor play 113. The developmental task for the family of an adolescent is to: a. Allow the adolescent increasing freedom and responsibility 114. Which of the following diseases is not acquired trans placentally? a. Tuberculosis 115. A two-year-old has iron deficiency anemia and has not had an increase in hemoglobin for the past three months despite Iron supplementation. The nurse practitioner should: a. Consider referral to a hematologist for further assessment 116. A 38-year-old penicillin allergy patient has folliculitis on A bearded part of his face. The nurse practitioner prescribes erythromycin and tells the patient: a. Erythromycin may upset his stomach, so it should be taken with food. 117. A 45-year-old male is diagnosed with hypertension. His blood pressures have been gradually increasing overtime, with a reading today of 152/98. Diagnostic studies are normal. The most likely diagnosis for this patient is: a. Essential hypertension 118. Which of the following signs and symptoms are typical of hypothyroidism? a. Constipation119. The most commonly recommended method for prostate cancer screening in a 55-year-old man is: a. prostate specific antigen (PSA) testing alone 120. An obese 43-year-old has recurrent superficial fungal skin infections over the past two years. Today, she presents with intertriginous candida. Her skin is macerated from frequent rubbing and scratching. The plan of care should include: a. Recommended screening for diabetes myelitis and HIV infection 121. If 37-year-old female patient with A history of a single episode of depression and frequent complaints of PMS is being treated for hypothyroidism. Today she complains of poor concentration and fatigue. The nurse practitioner should: a. Question her further 122. Which of the following drugs has a beneficial effect on benign prosthetic hyperplasia? a. Doxazosin (Cardura) 123. A patient that has 20/80 Visual acuity can see: a. At 20 feet what a person with normal vision can see it 80 feet 124. The goals of treatment for patients with alcohol abuse disorder are: a. Abstinence or reduction in use, relapse prevention, and rehabilitation 125. The nurse practitioner correctly teaches the patient with diabetes mellitus that if blood glucose is less than 100mg/dl (5.6 mmol/L) prior to exercise, the patient should first consume 15 g of carbohydrates, such as: a. Once slice of bread 126. Characteristics of prescription and OTC drug use in the elder population include all of the following except: a. Increased tolerance to drug effects 127. Acute lymphocytic leukemia (ALL) is usually diagnosed by: a. A bone marrow examination 128. Subsequent to successful completion of the nurse practitioner certification exam, the candidate is considered to be: a. CERTIFIED129. An obese hyperlipidemic patient, newly diagnosed with type II diabetes mellitus has fasting blood glucose values 180 to 250 mg/dL (10.1-14 mmol/L). What is the most appropriate initial treatment to consider? a. A sulfonylurea and /or metformin (Glucophage-XR) 130. All of the following are typical of toddlers except: a. Increased appetite 131. Interest practitioner has treated an infant for the last three weeks for thrush with nystatin (Mycostatin) oral suspension. What diagnostic test could the nurse practitioner platform which might quickly confirm a diagnosis of oral candidiasis? a. A potassium hydroxide (KOH) slide preparation 132. Which of the following physical findings is consistent with a diagnosis of Beta-thalassemia major? a. Bronze skin color 133. An adult female patient is seeking information about her ideal weight. She is 5’7” tall using the” height weight formula”, what is her ideal body weight? a. 135 pounds 134. The most appropriate therapy for an elderly patient with pernicious anemia is: a.Intramuscular injection of vitamin B 12 135. Which of the following patients is most likely to have a diagnosis of type II diabetes mellitus? a. A 49-year-old male with impotence and HBA1C 10.0% 136. The recent Beta-adrenergic blockers should be avoided in patients with diabetes is because they may: a. Mask symptoms of hypoglycemia 137. The nurse practitioner is initiating levothyroxine (Synthroid) for primary hypothyroidism in a 71-year-old female. The usual starting dose of levothyroxine is 50 to 100 micrograms per day. What would be the most appropriate initial therapy in this patient? a. 25 µg per day 138. Nurse practitioner is evaluating a 35-year-old female nurse. She has a history of hospitalization for hepatitis B infection two years ago. Her laboratory testsdemonstrate positive HBsAg/ surface antigen. The nurse practitioner would most likely diagnose: a. A chronic hepatitis B infection 139. Phalen’s test, 90° wrist flexion for 60 seconds, reproduces symptoms of: a. Carpal tunnel syndrome/ I think the question states what will show in a person who has Carpal tunnel syndrome, and the answer is a positive phalens and tinnels sign 140. According to the American nurses Association (ANA), the role of the nurse practitioner who provides primary healthcare is: a. Direct nursing care role 141. Health maintenance organization (HMOs) and preferred provider organizations (PPOs) are: a. Managed care systems 142. A nurse practitioner has been sued by a patient for malpractice. The nurse practitioner is considered to be a: a. Defendant 143. A young couple is being seen by the nurse practitioner for preconception counseling. They Express the wish for pregnancy within the next three months and are very eager to know what they can do now to “make the baby as healthy as possible”. Which of the following should the nurse practitioner in courage to decrease the chance of neural tube defect in the fetus? a. Folic acid 0.4 mg daily 144. An active 82-year-old male in good health complains “I don’t see as well as I used to and my eyes are very sensitive to glare.” His near and distant visual acuity is diminished, and he has a bilateral white pupillary reflex. The most likely diagnosis is: a. Cataracts 145. The Papanicolaou (PAP) smear report on a 36-year-old female patient indicate atypical squamous cells of undetermined significance (ASCUS). Which of the following constitutes appropriate care for this patient? a. Repeat the Pap smear in four months146. A 23-year-old female college student is being evaluated by the nurse practitioner for immunization status. She has documentation of completion of IPV, DTaP, and MMR series. She states, “I got a shot when I was 12 years old, but none since then.” Which vaccines should she received today? a. Td and HBV 147. A 20-month-old Child has been brought to the clinic after having a simple febrile seizure. The most appropriate intervention is to: a. Educate the patient about febrile seizures and first aid 148. A 75-year-old female complains that she awakens 3 to 4 times each night sensing bladder fullness, but is unable to “hold it” until she can get seated on the bathroom toilet. This type of urinary incontinence is termed: a. Urge incontinence 149. A one-month old presents with reported recurrent diarrhea, screaming, and drawing up of the legs followed by periods of lethargy. On physical examination, A” sausage-like” mass is palpated in the upper right quadrant of the distended abdomen. Which of the following is the most likely diagnosis? a. Intussusception 150. A critically ill patients states, “my family is well provided for”. The nurse practitioner must interpret this meaning as the patient: a. Wishes to discuss the topic of death 151. A 46-year-old female has hypertension and is well managed on propranolol (Inderal). Which of the following is a beneficial secondary effect of this drug? a. Migraine prophylaxis 152. During pregnancy, it is important to be physically active. The nurse practitioner should encourage her pregnant patient who has a very sedentary lifestyle to: a. Start walking daily and slowly build up distance 153. A 19-year-old pregnant patient, at 20 weeks gestation, complains of pain in the right lower quadrant. She is afebrile and denies nausea and vomiting. The most likely diagnosis is: a. Round ligament pain 154. Which of the following is not covered by Medicare part B?a. Services and short stay skilled nursing care facilities 155. Which of the following patient characteristics are associated with chronic bronchitis? a. Overweight, cyanosis, and normal or slightly increased respiratory rate 156. Which of the following patients would appropriately be diagnosed with isolated systolic hypertension (ISH)? a. 69-year-old female with a blood pressure of 156/86 157. Which of the following drugs is considered safe for use during pregnancy? a. Miconazole (Monistat) cream 158. A 44-year-old patient complains of stiffness and soreness in his hands, hips, and knees. There is noticeable PIP and DIP joint enlargement in his hands. The nurse practitioner suspects arthritis. All of the following questions are helpful in differentiating between rheumatoid arthritis (RA) and osteoarthritis (OA) except: a. Have you noticed decreased joint movement or flexibility? 159. A 55-year-old male patient presents with dysuria, urgency, peroneal pain, and temperature of 10 1°F. What is the most likely diagnosis? a. Prostatitis 160. The nurse practitioner receives a call from a 23-year-old patient who is 10 weeks pregnant. She reports that a few days after spending the day babysitting a friend’s toddler, the toddler develops a rash. The nurse practitioner would be least concerned if the patient was exposed to: a. Hep A 161. For the general adult population, total dietary fat intake should be no more than what percent of total calories? a. 30% 162. The mechanism for handling first contacts into the healthcare system and providing a continuum of care, evaluation and management of systems maintenance of health, and appropriate referrals is called: a. Case management 163. The four classic features of Parkinson’s disease are: a. Tremor at rest, rigidity, Bradykinesia, and postural disturbances164. The most common symptoms of transient ischemic attack (TIA) include: a. Weakness and extremity, abruptly slurred speech or partial loss of vision, and sudden gate changes 165. A 15-year-old patient has a complaint of vaginal discharge. She is sexually active with multiple partners. Which of the following symptoms should leave the nurse practitioner to suspect pelvic inflammatory disease (PID)? a. Cervical motion tenderness (CMT) 166. The nurse practitioner wants to assess intactness of a patient’s cerebellar function. Which of the following clinical test will provide information relative to cerebellar function? a. Romberg 2. A patient presents with distorted area and a blind spot in the visual field. The painless condition was first noticed on awakening this morning. The nurse practitioner would most appropriately include all of the following in the plan of care except: a. Refer the patient to an ophthalmologist if the condition does not get better within 24hrs. 3. Which murmur is associated with radiation to the neck? a. Aortic stenosis 4. The correct order of physical examination of the abdomen is: a. Inspection, auscultation, percussion, palpation 5. What disease process is Bouchard’s nodes most commonly associated with? a. Osteoarthritis (OA) 6. A patient has Kawasaki syndrome. Which characteristic would be unusual? a. Age >15years 7. The nurse practitioner should instruct the mother of an infant with thrush to: a. Sterilize pacifiers and bottle nipples 8. A 24-year-old female taking oral contraceptives has missed her last 2 pills. What should the nurse practitioner advise her to do? a. Double today’s dose and tomorrow’s dose and use a barrier method the rest of this month9. A 65-year-old diabetic patient has newly diagnosed peripheral artery disease in her lower extremities. She is exhibiting symptoms of this disease. In assessing this patient, the nurse practitioner would expect to find: a. Dependent rubor 10. A patient recovering from a recent stroke is starting anticoagulant therapy. The nurse practitioner should teach the patient to avoid all of the following except: a. Milk and milk products 11. The anatomical site currently believed to be the best location for subcutaneous insulin administration is: a. Abdomen 12. A patient presents with periorbital erythema and edema, fever, and nasal drainage. The nurse practitioner should: a. Start aggressive antibiotic therapy 13. A nurse practitioner is examining a 2-month-old infant. What question is most important to ask about the baby? a. Was the infant full-term at birth 14. Which of the following patients is least likely to develop osteoporosis secondary to a comorbid condition? a. A 28-year-old female with type II diabetes 15. A 72-year-old man has chronic prostatitis. What is the initial drug treatment of choice for this patient? a. Ciprofloxacin 16. Practitioners working with physically active girls need to be aware of the “female athlete triad” in order to develop an effective plan for prevention, recognition, and treatment. The component of the female athlete triad are: a. Eating disorder, amenorrhea, and osteoporosis 17. Which of the following drug classes may potentiate hyperkalemia in a patient taking potassium-sparing diuretic or potassium supplement? a. Angiotensin converting enzyme inhibitor 18. Salicylates and other nonsteroidal anti-inflammatory agents have all of the following actions except:a. Anti-emetic effects via suppression of gastric acid secretion 19. The obesity associated with type II diabetes mellitus is: a. Defined as BM I 30% or greater 20. Retinopathy specifically associated with poorly controlled hypertension is manifest clinically as: a. AV nicking 21. Electro cardiogram markers of hypokalemia include all of the following except: a. Peak T waves 22. Ideally, antepartum care should begin: a. With preconception counseling 23. A nurse practitioner is providing guidance to a newly diagnosed diabetic patient who is being treated with insulin. The nurse practitioner would be correct to tell the patient to self-treat signs and symptoms of hypoglycemia with: a. 15 g of sugar, or 5 Lifesavers 24. An 18-year-old college student lives in the dormitory. He has been treated for scabies infestation with permethrin (Nix). He was asymptomatic for two weeks but now complaints again of itching and skin burrows. How should the nurse practitioner proceed? a. Retreat him with that permethrin and have him launder all of his bedding and clothing 25. What is the most common causative pathogen found in cystitis, pyelonephritis, and prostatitis? a. Escherichia coli (E.Coli) 26. A 26 -year-old female presents with vaginal itching and a malodorous vaginal discharge. What prep is positive for budding hyphae and flagellated protozoa a negative for wbc’s and clue cells. KOH is positive. Based on these lab and microscopic findings, the most likely diagnoses are: a. Trichomonas and vaginal candidiasis 27. Which of the following maybe caused by long-term glucocorticoid treatment? a. Cushing’s syndrome 28. The most appropriate treatment for a child with mild croup is:a. Cool mist vaporizer 29. A patient states “I just found out my old boyfriend died of AIDS.” She is worried that she has contracted illness and wants to be tested. The nurse practitioner correctly offers this patient the: a. ELISA as the initial screening test 30. A 40-year-old patient presents with a hordeolum. The nurse practitioner teaches the patient to: a. Apply a topical antibiotic and warm compresses 31. When counseling a woman who is breast-feeding her six month old infant, the nurse practitioner should recommend a caloric intake over her pre-pregnancy requirement of: a. 500 kcal/ day 32. A seven-year-old male presents with a painless limp, antalgic gait, muscle spasm, mildly restricted hip abduction and internal rotation, proximal thigh atrophy, and slightly short stature. Most likely diagnosis is: a. Legg-Calve-Perthes disease 33. Among adolescents in the US, the greatest known risk factor for contracting hepatitis B is: a. Heterosexual activity 34. A 45-year-old obese premenopausal female complains of indigestion, flatulence, RUQ and epigastric “crampy pain” that radiates to the right scapula. Symptoms are exacerbated by a high fat meal. What is the most likely diagnosis? a. Chronic cholecystitis 35. A 51-year-old postmenopausal female, request guidance regarding osteoporosis risk. The nurse practitioner would be correct to recommend all of the following except: a. Weight loss 36. A 40-year-old female with a history of frequent sun exposure presents with a multi colored lesion on her back. It has irregular borders and is about 8 mm in diameter. What should the nurse practitioner suspect?a. Malignant melanoma/ I think the question asks what do you do? I think I put refer to surgeon or derm for further tx 37. A 60-year-old male patient with multiple health problems presents with a complaint of erectile dysfunction. Of the following, which medication is most likely to be causing the problem? a. Methyldopa (Aldomet)/ the question asks what drug class may cause ED 38. On examination of the skin a 65-year-old male, the nurse practitioner notes erythematous eruptions with scaly and flaky, but oily, yellowish plaques over the nasolabial fold, scalp, and area of the eyebrows. The patient reports that condition is always present and exacerbated by stress. What is the most likely diagnosis? a. Seborrheic dermatitis 39. A 21-year-old college student presents to the student health center with copious, markedly purulent discharge from her left die. The nurse practitioner should suspect: a. Gonococcal conjunctivitis 40. A 32-year-old mother and her 10-year-old child each have a 10 cm round, reddened patch on the trunk. There is Central clearing of the lesion. Different lesions run parallel to each other in a Christmas tree pattern. The mother has been treating the “ringworm” with an antifungal cream for seven days without success. The nurse practitioner’s best response is: a. No medication will help this to clear. In must clear on its own and may take 4 to 8 weeks 41. The nurse practitioner is disgusting contraception options to be used during lactation with a 24-year-old pregnant patient. Which of the following methods of contraception would not be appropriate for use while breast-feeding? a. A low dose oral contraceptive 42. The hyperinsulinemia and insulin resistance associated with the diagnosis of syndrome X causes all of the following except: a. tachyarrhythmias and angina 43. A 49-year-old Hispanic female has a blood pressure of 145/95 mmHg during a routine annual evaluation. She has no previous history of hypertension. She takes a Statin for dyslipidemia. How should the nurse practitioner proceed with this patient?a. The patient’s blood pressure should be checked in about 2 to 4 weeks 44. A 16-year-old male presents with mild sore throat, fever, fatigue, posterior cervical adenopathy, in Palatine petechia. Without a definitive diagnosis for this patient, what drugs would be the least appropriate to prescribe? a. Ampicillin 45. What is an appropriate drug for prophylactic treatment of migraine headaches and a 21-year-old female? a. Propranolol (Inderal) 46. Introduction of solid foods to infants should begin: a. No earlier than 4 to 6 months of age, beginning we serial, then fruits, one at a time 47. A 16-year-old sexually active student presents with complaints of a greenish gray frothy vaginal discharge and vaginal itching. The nurse practitioner should suspect: a. Trichomoniasis 48. All of the following principles apply to drug therapy for elders except: a. Encourage the elderly to continue to self-medication administration to facilitate independence and self-sufficiency 49. Criteria for the diagnosis of Alzheimer’s disease include: a. Insidious and progressive decline of cognitive functions 50. A 10-year-old presents with hematuria, periorbital edema, and elevated blood pressure. He has a history of streptococcal pharyngitis two weeks earlier. What is the most likely diagnosis? a. Acute glomerulonephritis 51. After being thoroughly examined, a child is diagnosed with recurrent abdominal pain. Patient and parent counseling should include: a. Acknowledgment that the pain is real, although there is no physical cause 52. Eligibility requirements for the ANP or FNP exam include all of the following except: a. A master’s degree in nursing or an area 53. Which of the following can result from chronic inflammation of a Meibomian gland? a. A chalazion54. The nurse practitioner is caring for a 19 year old female college student with iron deficiency anemia secondary to heavy menstrual bleeding. An appropriate initial treatment for this patient is: a. Oral ferrous sulfate 55. A 23-year-old female presents with scaly hypo pigmented macular lesions on her trunk, shoulders, and upper arms. The lesions flouresce under Wood’s lamp. Appropriate treatment for this condition is: a. Apply selenium sulfide (Versel) lotion and allow to dry for 10 minutes; then rinse off 56. P6 do-year-old male patient with a past history of glaucoma and frequent sinusitis presents today with hypertension. On his last two visits to the clinic, his blood pressures were 150 to 160 / 90 to 98. The nurse practitioner decides to treat the hypertension with long acting propranolol (Inderal). Before prescribing it the nurse practitioner should ask: a. What other medications have been prescribed for him 57. Once a competent adult patient is identified as a victim of domestic violence, the best plan is to: a. Suggest resources that will help the victim to develop survival skills 58. A patient has been taking enalapril (Vasotec) for a period of 14 months to treat heart failure. Despite feeling well otherwise, she has developed you dry, nonproductive cough. The nurse practitioner knows that: a. The cough should subside 1 to 4 days after discontinuing enalapril 59. A 20-year-old female patient presents to the emergency department with lower abdominal pain, moderate vaginal bleeding, and right shoulder pain. Her blood pressure is 90/50 mmHg, pulse is 120, respirations 20, and temperature 98°F. She has a history of pelvic inflammatory disease (PID). Her urine pregnancy test is positive. Which of the following is the most likely diagnosis? a. Ectopic pregnancy 60. A 27-year-old female patient with epilepsy is well controlled with phenytoin (Dilantin). She requests information about contraception. The nurse practitioner should instruct her that: a. The effectiveness of an oral contraceptive may be reduced61. Microalbuminuria is a measure of: a. Protein lost in the urine 62. What problem would be least expected any pregnancy complicated by gestational diabetes? a. Placenta previa 63. Which of the following findings would raise the nurse practitioners’ suspicion of bulimia and a 17-year-old female? a. Scars on her knuckles 64. A first-time mother asks the nurse practitioner how much her baby will grow in the first three months of life. The nurse practitioner explains that the normal baby will grow: a. 1/2 to 1 ounce per day 65. Which of the following has been shown by research to be most influential in promoting smoking cessation in the adolescent population? a. Cigarette smoking contributes to yellow teeth, bad breath, and decreased capacity to perform sports activities. 66. A 23-year-old college student presents to the campus health clinic with a generalized rash on her trunk. The lesions are oriented in a Christmas tree pattern and I’m widely pruritic. She describes a “round ringworm” that appeared in about one week ago, but now has disappeared. What is the patient’s diagnosis and how should she be treated? a. Pityriasis rosea: treat it with a topical anti-pruritic 67. Healthy people 2010 published by the US Department of Health and Human Services: a. Is a set of national health objectives designed to improve the overall health of people and communities. 68. Which case epistaxis should concern the nurse practitioner most? a. A child under two years of age 69. A very active 35-year-old male has painful hemorrhoids, but he does not want hemorrhoidal surgery at this time. His diet has been indiscriminate as his job requires frequent travel. The most appropriate recommendation is for him to select foods that are: a. High in fiber such as bran, complex carbohydrates, and fresh fruit70. The CDC recommends the pneumococcal polysaccharide vaccine for: 1. Adults > 60 years of age. 2. Asplenic person > or equal to two years of age. 3. HIV it affected individuals and diabetic adults. 4. Patients with CHF, or chronic liver disease. a. 2,3,4 71. A patient has been diagnosed with hypothyroidism in thyroid hormone replacement therapy is prescribed. When should the nurse practitioner check the patients TSH? a. I think the answer is four -Six weeks 72. It is recommended that the therapeutic management of children with juvenile rheumatoid arthritis (JRA) should include: a. Ophthalmologist examination at least annually 73. The child with type I diabetes mellitus brings in a glucose diary indicating consistent morning hyperglycemia. How can the nurse practitioner differentiate the Somogyi effect from Dawn phenomenon? a. Instruct a parent to monitor the blood glucose at 3 AM 74. A 16-year-old presents for a sports physical for football. The nurse practitioner auscultates a diastolic murmur. It is a grade II/VI. He has no hx of a murmur. The patient denies symptoms. What is the most appropriate action for the NP? a. Refer the patient to a cardiologist. 75. Which age group is at higher risk for testicular cancer? a. 20-40 years-of-age 76. At what age is screening most helpful in detecting scoliosis? a. 12-14 years 77. What disease process are Heberden nodes most commonly associated with? a. Osteoarthiritis (OA) 78. A pre-adolescent male expresses concern about his physical growth and development as compared to girls his age. The NP correctly tells his that pubertal growth spurt occurs about: a. 1 year earlier in boys79. The mother of a 3-year-old male asks about her son’s nocturnal enuresis. Which of the following responses is appropriate? a. Generally, nocturnal enuresis is not considered to be a problem before 6-years of age. 80. Persons with thalassemia should avoid: a. Iron replacement therapy 81. When examining a pregnant patient, where should the fundal height be at 22 wks.? My question asked about a 20 week old pt a. Above the umbilicus 82. The nurse practitioner preforms a routine physical examination of a 3-years old. A hard, painless mass is palpated in the abdomen, along with the lymph node enlargement and lower limb paresis. Blood imaging studies provide markers for neuroblastoma. What information is correct to give the parents? a. The diagnosis must be confirmed by tissue biopsy or by bone marrow aspiration plus urine or serum catecholamine levels. 83. Which of the following is a “red flag” indicating possible developmental delay? a. A 36 month old has a 50 word vocab and %0% of speech is intelligible. 84. Babies should begin oral iron supplementation at a. 4-6 months old 85. A patient with moderate persistent asthma will probably be most effectively managed with daily; a. Inhaled steroids and long acting bronchodilators. 86. A 17-year old presents with a wound received while building a fence. He has completed a primary vaccination series and he had a Td booster 3 years ago. Does he need a Td booster today? a. No, expert opinion supports vaccination after contaminated wound when more than 5 years have elapsed since last Td vaccine. 87. The infant 1 to 6 months of age with a diagnosis of developmental hip dysplasia is correctly treated with: a. The Pavlik Harness88. A 60-year old female patient complains of sudden onset unilateral, stabbing, surface pain in the lower part of her face lasting a few minutes, subsiding and then returning. The pain is triggered by touch or temp extremes. Physical exam is normal. The patient had a negative dental evaluation. CT scan is normal. Erythrocyte sedimentation rate is normal. Which of the following is the most likely diagnosis? a. Trigeminal neuralgia 89. Which of the following demonstrates that a 30-year old male has successfully achieved the developmental task of young adulthood? a. Is married with 2 children 90. Diagnostic radiological studies are indicated for low back pain: a. When there is suspicion of a space-occupying lesion, fracture, cauda equina, or infection 91. A healthy 50-year-old patient presents to the clinic for a routine physical examination. He has no significant personal or family history of clinical heart disease. What should be part of his CV assessment? a. blood pressure only 92. Patients diagnosed with polycystic ovarian disease are at increased risk for developing: a. Diabetes mellitus 93. Why is it important that a post-menopausal woman with an intact uterus receive combined estrogen-progestin hormone replacement therapy rather than estrogen alone? a. Prolonged use of unopposed estrogens increased the risk for endometrial cancer 94. A professional liability insurance policy that provides coverage for injuries arising out of incidents occurring during the period the policy was in effect, even if the policy subsequently expires, or is not renewed by the policy holder, is termed a(an): a. Occurrence based policy 95. When prescribing an iron supplement to a child it is most important to tell the mother: a. To keep this and all other medications out of reach of children.96. An 87-year old was placed on low dose amlodipine (Norvasc) for the treatment of hypertension and angina. She takes an ASA daily, but, takes no other medications. What side effects might be expected from taking amlodipine? a. Orthostatic hypotension 97. An 18-year old female applying for college admission presents to the health clinic because evidence of rubella vaccination is required for admission. She says” I don’t ever remember getting that shot” she has a negative serologic evidence of rubella antibody. The nurse practitioner should: a. Administer the vaccination after negative pregnancy test and advise the patient that she must not get pregnant for 28-days. 98. An overweight male complains of low back pain after helping to move a refrigerator over 2 days ago. To avoid future problems, the nurse practitioner should teach him: a. Exercises to strengthen his lower back 99. Infants with celiac disease (gluten enteropathy) are at risk for multiple complications. The most urgent complication of the disease is: a. Intussusception or volvulus 100. An elderly patient’s digoxin level has decreased to 0.4 ng/dl and she has begun to have atrial fibrillation again. For the past 14 months, her digoxin level had been well controlled when her digoxin level was 1.4 to 1.5 ng/dl. What could not have accounted for this change in digoxin level? Normal digoxin level is (0.8-2 ng/dl). a. Decreased creatinine clearance 101. An 83-year-old man has a resting tremor. What disease process is this type of tremor most commonly associated with? a. Parkinson’s disease 102. A 70-year-old patient presents with left lower quadrant abdominal pain, markedly tender palpable abdominal wall, fever, and leukocytosis. Of the following terms, which correctly describes the suspected condition? a. Diverticulitis 103. A 16-year-old sexually active female presents to the clinic. She has never had vaccinations for hepatitis A or B, she has had one MMR immunization, and her lasttetanus vaccination was four years ago. Which vaccination would be contraindicated without further testing? a. MMR 104. Patients with human immunodeficiency virus (HIV) are at increased risk for pneumocystis carinii pneumonia (PCP) which is associated with significant morbidity and mortality. For this reason, prophylaxis is recommended. Which of the following is inappropriate drug for PCP prophylaxis? a. Trimethoprim-sulfamethoxazole Bactrim) 105. A 15-year-old female reports she is a strict vegetarian. Which of the following is the most appropriate response by the nurse practitioner? a. Include a wide variety of legumes, enriched grains, nuts, and seeds in your diet 106. Right sided heart failure is characterized by all of the following clinical findings except: a. A fourth heart sound 107. The nurse practitioner diagnoses eczema on the cheeks of a two-year-old female. The patient’s mother explains that she is embarrassed to take the child in public because her face is so red, dry, and crusted. She asked for a “strong” cortisone cream so the eczema will clear rapidly. The nurse practitioner knows that: a. A high potency cortisone cream may cause atrophy, telangiectasia, purpura, or striae if used on the face 108. A patient presents with a normal complete blood count and a positive stool for occult blood. The nurse practitioner decides to repeat this stool for occult blood after having the patient eliminate certain foods from the diet for 72 hours. Which of the following should be included in the list of foods to omit? a. Steak 109. Which term most accurately describes the prostate gland of a patient with prostate cancer? a. Hard 110. A 30 year old female patient presents to the clinic with heat intolerance, tremors, nervousness and weight loss inconsistent with appetite. Which choice below would be most helpful in identifying the etiology of this patient’s complaints?a. A serum thyroid stimulating hormone level 111. Diagnosis of systemic lupus erythematosus (SLE) is made: a. Considering symptom complex with confirmation by laboratory tests 112. A three-year-old female with a palpable right upper quadrant abdominal mass, anemia, and fever is being evaluated for Wilms’ tumor. Which of the following diagnostic test would be most useful? a. Abdominal ultrasound 113. In July 1996, the advisory committee on immunization practices of the centers for disease control and prevention issued which of the following recommendations for the use of the live attenuated varicella vaccine? a. Varicella vaccine has been approved for routine use and all healthy children age 12 months to 12 years 114. Advances in obstetrics in neonatal care have: a. Had no effect on the incidence of cerebral policy 115. What is the most common chronic condition in the elder population in the United States? a. Arthritis 116. In comparing sensitivity to specificity, sensitivity refers to a: a. True positive 117. The best way for a pregnant woman to avoid injury to herself and her fetus in the event of a motor vehicle accident is to: a. Use both the lap and shoulder restraint 118. A 25 year old female patient presents for routine well woman exam. On physical examination, the nurse practitioner notes a scant nipple discharge, absence of a palpable mass, and absence of lymph node enlargement. Which of the following is the most likely diagnosis? a. Intraductal papilloma 119. A 22-year-old patient has a single, nontender, freely movable lump in her right breast. She denies any nipple discharge. Which of the following diagnosis is this clinical presentation most consistent with? a. Fibroadenoma120. The most effective treatment for anemia of chronic disease is: a. Treatment of the underlying disease 121. A 65 year old African-American patient was screened at his local church for “High cholesterol”. His total cholesterol (nonfasting) was 215mg/dl. What action below is most appropriate for the nurse practitioner? a. Recommend a fasting lipid profile 122. A 72-year-old female patient reports a six month history of progressively more swollen and painful distal interphalangeal joints of one hand. There are no systemic symptoms but the erythrocyte sedimentation rate, antinuclear antibody, and rheumatoid factor are all minimally elevated. What is the most likely diagnosis? a. Osteoarthritis 123. When discussing treatment options with the patient, the nurse practitioner would be correct to say that surgery can be curative for: a. Ulcerative colitis 124. A 26-year-old female was informed that she is four weeks pregnant. Because of her history of smoking half a pack of cigarettes per day, the nurse practitioner explains that smoking during pregnancy increases the risk of her infant being born: a. Prematurely 125. A 12-year-old presents with ear pain of 36 hours duration. The nurse practitioner diagnosis acute otitis media because the: a. Bony landmarks are obscured in the Tympanic membrane is mildly erythematous, doll, and immobile 126. Which of the following is an example of secondary prevention? a. Annual mammography for women age 40 years and older 127. A 17-year-old female presents with a painful the secular lesion on her vulva. Which of the following would be the most definitive diagnostic test? a. Tzanck prep 128. The nurse practitioner correctly diagnoses a second degree burn injury which is described as: a. Partial thickness, with involvement of the dermis and epidermis, edema, and vesicles129. Long-range disease prevention programs are most likely to be successful if: a. The programs are shaped to fit the cultural and health profiles of the population 130. Salmeterol (serevent) in combination with an inhaled steroid is prescribed for a patient with moderate persistent asthma. What is the most important teaching point about salmeteral? a. It is not effective during an acute asthma attack 131. A 66 year old patient exhibits sudden onset of fluctuating restlessness, agitation, confusion, and impaired attention. This is accompanied by visual hallucinations and sleep disturbance. What is the most likely cause of this behavior? a. Delirium 132. A 15 year old pregnant patient presents for her first prenatal visit. On physical examination, her uterus is approximately 24 week gestational size. She does not know when her last menstrual period was nor does she know when she might have conceived. Gestational age for this patient can most accurately be assessed by: a. Ultrasonography 133. Which of the following is the most important diagnosis to rule out in a patient with acute bronchitis? a. Pneumonia 134. Upon ophthalmoscopic examination of a 78-year-old patient, the nurse practitioner observes dark spots against a red retina. What the diagnosis is this finding most consistent with? a. Cataract 135. A prenatal patient is in the 10th week of pregnancy. Rubella tighter was 1:18 on her first prenatal visit, at six weeks gestation. She believes she was exposed to rubella at yesterday and asks how this will affect her pregnancy. Which of the following responses by the nurse practitioner is appropriate? a. The pregnancy will be unaffected. You are immune to Rubella 136. Herniated nucleus pulposus typically presents clinically by all of the following except: a. Listing away from the unaffected side toward the affected side.137. The nurse practitioner correctly teaches an insulin-dependent patient the signs and symptoms of early hypoglycemia, which include: a. Blurred vision, pallor, and perspiration 138. The most reliable indicator of neurological deficit when assessing a patient with acute low back pain is (are): a. Decreased reflexes, strength, and sensation in the lower extremities 139. An 86-year-old Caucasian male requests screening for prostate cancer. What is the best approach to his request? a. Give him objective information about the potential benefits of early detection and treatment. 140. In order to decrease death from lung cancer: a. Patients should be counseled to quit smoking 141. Acute rheumatic fever is diagnosed using the modified Jones criteria. Which is not a major criteria in the Jones system? a. Jaundice 142. A 50-year-old male is scheduled for his annual wellness visit. The nurse practitioner would appropriately recommend all of the following except: a. Monthly self-testicular examination and annual clinical testicular examination 143. Considering mortality statistics for the adolescent age group, education targeted toward this group should first focus on: a. Alcohol abuse 144. Licensure is: a. Used to establish minimal competence 145. A nurse practitioner is holding a prenatal nutrition class for a group of parents. Considering knowledge about cultural variations, which of the following women may be at increased risk for inadequate calcium intake? a. A 27 year old native American 146. An 80-year-old Caucasian female has heart failure. Which symptom below is an early indication of failure? a. Weight gain147. An elderly patient presents with a great white ring around the periphery of the iris. This is probably: a. A normal variant associated with the aging process. 148. A 73 year old patient presents with a very sore, glossy, smooth, beefy-red tongue. This clinical presentation most likely reflects a. Pernicious anemia, due to insufficient intrinsic factor 149. Which of the following statements about medications in a 45 year old patient with diabetes is correct a. Daily aspirin therapy should be recommended for the secondary prevention of cardiovascular disease 150. The most appropriate treatment for a child with mild croup is a. A cool mist vaporizer 151. Which of the following statements about tuberculosis (TB) is accurate a. A positive skin test (PPD) reaction indicates exposure to tuberculosis, but follow up confirmatory cultures are required for a diagnosis 152. A 65-year-old diabetic patient has newly diagnosed peripheral artery disease in her lower extremities. She is exhibiting symptoms of this disease. In assessing this patient, the NP would expect to find a. Diminished pedal pulses 153. A 12-month-old has conjunctivitis in his right eye with a mucopurulent discharge. The mother asks if the child can forego the antibiotic eye drops because he doesn’t take drops put in his eyes. The NP replies that a. If untreated, conjunctivitis may permanently damage the cornea 154. A 30-year-old female patient presents to the clinic with heat intolerance, tremors, nervousness and weight loss inconsistent with appetite. Which choice below would be most helpful in identifying the etiology of this patient’s complaints a. A serum thyroid stimulating hormone (TSH) level 155. A new patient presents to the nurse practitioner clinic stating she wants a second opinion. She started prophlthiouracil (PTU, prophyl-thyracil) 75mg 3 times a day a week ago as therapy for newly diagnosed Grave’s disease. She still feels irritable and jittery. How should the NP responda. Inform the patient that improvement requires 2-3 weeks 156. Which of the following is not true regarding the diagnosis of scoliosis in children a. Kyphosis in the adolescent indicates scoliosis 157. Which of the following descriptions of the Denver II Developmental screening test is most accurate a. Applicable to children from birth to 6 years; evaluates 4 major categories of development 158. A young female reports onset of right flank pain 2 days ago that is now severe. Last night she discovered a “burning rash” in the same area. The NP identifies popular fluid filled lesions that are confluent and follow a linear distribution along the t-8 dermatome. The NP would appropriately order A. An oral antiviral 159. A 28 year old pregnant woman reports a history of smoking 1 pack of cigarettes per day. The NP is correct when she explains to the patient that cigarette smoking could be associated with a. Sudden unexpected infant death SID 160. Which of the following patients most warrants screening for hypothyroidism a. An elderly female with recent onset of mental dysfunction 161. Simple cases of impetigo and folliculitis are usually successfully treated with a. Topical antibiotics 162. When examining a 5 year old the NP knows that this is an appropriate age to teach the child about a. Not allowing any one to touch his private parts without permission 163. A 43 year old male presents with a large and painful furuncle, the 3rd one in the past 6 months. The NP should plan all of the following except a. Prescription of a prophylactic antibiotic 164. A 32 year old mother and her 10 year old child each have a 10cm round, reddened patch on the trunk. There is central clearing in the lesion. Different lesions run parallel to each other in a Christmas tree pattern. The mother has been treating the “ringworm” with an antifungal cream for 7 days without success. The NPs best response isa. No medication will help this to clear. It must clear on its own and may take 4-8 weeks 165. The result of the Weber tuning fork test is lateralization of sound to the right ear. This finding indicates a a. Conduction problem in the right ear 166. Which commonly used herbal remedy is not associated with anxiety and/or depressive symptom relief a. Ginkgo biloba 167. Which of the following patients is least likely to exhibit secondary hypertension a. A 62 year old with untreated depression 168. Which of the following must be present for the diagnosis of bacterial vaginosis a. Presence of clue cells 169. A 49 year old man sees the NP for the evaluation of a 2 mm macular lesion on his back. The lesion is brown with regular borders. The most appropriate action for the NP is to a. Reassure the patient that this lesion is not suspicious for pathology 170. The family of a 78 year old man moved him into an assisted living center because he can no longer be left at home alone. He is unable to toilet when asked to do so and he has had several episodes of incontinence. He has walked out of the facility twice and been unable to find his way back from 3 blocks away. On examination he is pleasant but mildly confused. Which of his medication is least likely contributing to his behavior a. Rampiril 171. A 4 year old female complains of leg pain at night which resolves by morning. This has lasted for the past 4 months. The NP should tell the patient’s mother that a. These are growing pains that last from 1 to 2 years. This is a common complaint in this age group 172. Which of the following diseases is not acquired transplacentally a. TB 173. A palpable thyroid nodule is benign. How does it feel on palpation a. Smooth174. Which term most accurately describes the prostate gland of a patient with prostate cancer a. Hard 175. A 10 year old female’s mean corpuscular volume (MCV) is 73 fl. What type anemia would the NP suspect a. Iron deficiency 176. What information should patients with diabetes and their families receive about hypoglycemia a. Hypoglycemia is serious, dangerous, and can be fatal if not treated quickly 177. The cornerstone of treatment for stress fracture of the femur or metatarsal stress fracture is a. Absolute rest from activities which may further stress the bone 178. Patients with benign prostatic hyperplasia (BPH) should be taught to avoid which of the following drug classes a. Tricyclic antidepressants 179. A mother who has diabetes mellitus wants to breast feed her infant. The NP should a. Support the mother’s decision and recommend more frequent monitoring of blood sugars 180. An important measure to prevent complications in children with sickle cell anemia is a. Staying well hydrated 181. A 32 year old male patient complains of urinary frequency and burning on urination for 3 days. Urinalysis reveals bacteriuria and positive nitrates. He denies any past history of UTI. The initial treatment should be a. Bactrim for 7-10 days 182. A 25 year old patient presents with complaints of pain and burning in the vulvar area. Upon examination, the NP notes 4 vesicles with an erythematous base arranged in a group on the patient’s labia majora. The most likely diagnosis is a. Herpes simplex II 183. Oral and parenteral contraceptive methodsa. Inhibit secretion of fsh 184. What is the recommendation of the US Department of Health and Human Resources regarding nicotine replacement therapy a. Encourage the use of nicotine replacement except in the presence of serious medical conditions 185. Therapeutic international normalized ratio (INR) for a patient taking warfarin for chronic atrial fibrillation is expected to be a. increased 186. A 48 year old female presents with a chief complaint of insomnia. On further investigation, she reports fatigue, nervousness, and agitation during the daytime and feeling hot most of the time. Which of the following would not be included in the differential diagnosis A. Parkinson’s disease 187. An 8 year old female has been brought to the nurse practitioner on 5 occasions in the past 9 weeks with a complaint of abdominal pain. The evaluation each time is negative, but her mother is convinced the child is truly experiencing pain. The most likely diagnosis is a. Recurrent abdominal pain 188. Patients with human immunodeficiency virus (HIV) are at increased risk for pneumocystitis carinii pneumonia (PCP) which is associated with significant morbidity and mortality. For this reason, prophylaxis is recommended. Which of the following is an appropriate drug for PCP prophylaxis A. Bactrim 189. The nurse practitioner would be correct to tell a 27 year old newly diagnosed hypertensive patient that a. Lifestyle modifications such as losing excess weight, exercising, limiting salt 190. Which of the following drug classes would be most effective in decreasing elevated triglyceride levels a. Statins 191. Which of the following drugs classes would be most effective in decreasing elevated triglyceride levelsa. Fibrates 192. A 52 year old female has a firm nontender one cm mass in the right lower quadrant of her breast. There are no palpable axillary lymph nodes. A mammogram the month before her examination was negative. The most appropriate nurse practitioner action today is to A. Schedule an ultrasound of the breast 193. A 26 year old female patient presents with cracked and sore nipples. She is breastfeeding her first child who is 4 weeks old. The nurse practitioner would be accurate to advise the patient to a. Apply a vitamin e moisturizing cream after each feeding and continue to breastfeed 194. A nurse practitioner is providing guidance to a newly diagnosed diabetic patient who is being treated with insulin. The nurse practitioner would be correct to tell the patient to self treat signs and symptoms of hypoglycemia with a. 15g of sugar, or 5 lifesavers 195. On examination a 67 year old patient is noted to have high tone hearing loss. This finding is consistent with what diagnosis a. Presbycusis 196. A patient with a past history of documented coronary arterial blockage less than 70% complains of chest pain several times per day (while at rest) which is relieved with nitroglycerin. What is the most appropriate initial action for the nurse practitioner? a. Refer to a cardiologist as soon as possible 197. A newborn with cyanosis secondary to a cardiac anomale probably has a. Tetralogy of fallot 198. A patient with diabetes mellitus calls the clinic reporting signs and symptoms of viral syndrome. She asks, “Should I skip my diabetes medicine until I get my appetite back?” The nurse practitioner reviews the sick day guidelines, which include all of the following except A. Hold all diabetes medications until eating habits have returned to normal 199. A 1 month old presents with reported recurrent diarrhea, screaming and drawing up of the legs followed by periods of lethargy. On physical examination, a “sausage like”mass is palpated in the upper right quadrant of the distended abdomen. Which of the following is the most likely diagnosis a. Intussescption 200. Upon ophthalmoscopic examination of a 78 year old patient, the nurse practitioner observes dark spots against a red retina. What diagnosis is this finding most consistent with a. cataract 201. Of the following, the patient who should be referred for periodic colonoscopy is the patient with a. Extensive ulcerative colitis of long duration 202. Adolescents are at increased risk for contracting the HIV due to a. Increased sexual experimentation 203. What is the most appropriate NP intervention for a 2 month old febrile infant who appears ill but has no evidence of specific infection upon examination a. Hospitalization, and blood, urine, and spinal fluid cultures, followed by antimicrobial therapy 204. Which immunizations are contraindicated in an immunodeficient individual? 1 varicella 2 ipv 3 mmr 4hbv a. 1,3 205. To differentiate testicular torsion from epididymitis, the np should order a. Doppler US 206. Expected findings associated with a diagnosis of folic acid deficiency include all of the following except a. Mcv <80 207. The drug of choice for acute episodes of gout is colchicine. What is the most common early sign of colchicine toxicity a. diarrhea 208. A 27 year old male athlete presents with complaints of exquisite pain over his right shin. He runs an average of 6 to 8 miles per day. He wonders if he has a right tibial stress fracture. Which response by the NP is not correct a. There is no radiological test which will identify a stress fracture209. A 25 year old married woman is being taught the natural family planning method of contraception by the nurse practitioner. Which of the following statements by the patient demonstrates her understanding of nfp a. Cervical mucus is clear and thin during ovulation 210. A 65 year old presents with a past medical history of coronary artery disease, temperature 101 degrees F, tachypnea 24 breaths per minute, and consolidation in the left lower lobe. A chest x-ray confirms left lower lobe pneumonia. What antimicrobial agents should the NP prescribe a. Levofloxacin 211. A 26 year old female was informed that she is 4 weeks pregnant. Because of her history of smoking half a pack of cigarettes per day, the NP explains that smoking during pregnancy increases the risk of her infant being born a. Prematurely 212. The NP strongly suspects hyperthyroidism in a 62 year old patient. Which of the following would not be an appropriate initial intervention a. Start levothyroxine 25mcg daily and reassess in 2 weeks 213. Which of the following oral medications should be avoided in a child under 8 years of age a. Tetracycline 214. The NP is holding a prenatal nutrition class for a group of patients. Considering knowledge about cultural variations, which of the following women may be at increased risk for inadequate calcium intake a. A 27 year old native american 215. A professional liability insurance policy that provides coverage for injuries arising out of incidents occurring during the period of policy was in effect, even if the policy subsequently expires, or is not renewed by the police holder is termed a. Occurrence based policy 216. A patient with uncontrolled primary hypertension presents with a complaint of frequent headaches. Headaches which may be attributed to the patient’s blood pressure elevation are typically described as a. Occipital presenting on awakening in the morning217. All patients with renal calculi should be taught that the best prevention is a. Increased fluid intake 218. Which of the following medications may predispose the patient to hypoglycemia a. Beta-adrenergic blockers 219. An urgent call is received from a patients wife who states that her husband is having chest pain unrelieved by 3 nitroglycerin tablets. The nearest hospital is 45 minutes away. Besides having the patient reach the hospital as soon as possible, which intervention can the NP suggest which might influence a positive outcome a. Have the patient chew an aspirin on the way to the hospital 220. A patient taking levothyroxine is being over-replaced. What condition is he at risk for a. osteoporisis 221. The NP suspects a TMJ disorder. As the patient slowly opens and closes the lower jaw, the NP palpates the temporomandibular joint bilaterally a. Anterior to the tragus 222. It is clear that in alzheimers disease, environmental forces as well as genetic factors exert considerable influence. One potential treatable risk factor currently receiving much attention is a. Estrogen deficiency 223. A mother presents with a 3 year old who has croup. The plan for home care would appropriately include a. A cool mist vaporizer 224. A patient presents with periorbital erythema and edema, fever, and nasal drainage. The NP should a. Start aggressive antibiotic therapy 225. Patients diagnosed with polycystic ovaridan disease are at increased risk of developing a. Diabetes mellitus 226. Which drug class is associated with elevated serum lipid levels a. Thiazide diuretics227. Which choice below is not a contraindication to receiving the diptheria, pertussis, and tetanus vaccine a. Current antibiotic use 228. A patient with hepatic cirrhosis presents with pedal edema. He reports that his wife prepares the same lunch for him every day :a ham and cheese sandwhich on white bread with mustard, a bowl of fresh vegetable soup a. Ham and cheese 229. What diabetic complications result from hyperglycemia? 1 retinopathy 2 hypertension resistant to treatment 3 peripheral neuropathy 4 accelerated atherogenesis a.1, 3, 4 230. The following are all important tests of visual acuity in infants and children up to 3 years of age except a. The Hirschberg test 231. Which of the following medication classes is considered first line therapy for patients who have heart failure a. ACE inhibitors 232. An adolescent goes to Colorado for a ski trip. He is unaccustomed to the high altitude and very dry air. He develops nose bleeds & visits the NP. What intervention is least effective a. Drink adequate water to promote hydration of the nares 233. Screening for increased intra-ocular pressure or early glaucoma is a. Best performed by an eye specialist 234. The most effective intervention to prevent stroke is a. Smoking cessation and treatment of hypertension 235. Which of the following should not be avoided when the patient has BPH a. Alpha adrenergic blockers 236. An appropriate initial treatment for benign positional vertigo is a. Meclizine 237. The NP palpates an enlarged right epitrochlear lymph node. The next action is to assess the a. Right forearm and hand238. Educating parents to avoid putting their children to bed with a baby body a. Helps prevent tooth decay 239. The major health risk associated with oral contraceptive therapy is a. Cardiovascular events 240. All of the following are associated with erectile dysfunction except a. Nsaids and oral hypoglycemic agents 241. At what age does vision normally become approximately 20/20 a. 6 years 242. The NP is performing a routine assessment of a 47 year old female who wants to lose weight. She has truncal obesity with relatively slender forearms and lower legs. Her BMI is 38. Upon review of her history, physical examination, and laboratory results, the NP diagnoses syndrome x. This diagnosis is based on the previous findings plus all of the following except a. Cardiac arrhythmias 243. The long term prognosis for a patient with diagnosis of dementia is a. Poor, because gradual deterioration of cognitive function, memory, judgment, and emotional stability is progressive and irreversible 244. A 75 year old patient is well controlled on timolol maleate for chronic open angle glaucoma. Prescribing propranolol for this patient may precipitate a. Bradycardia 245. Which of the following is least likely to be the cause of ophthalmia neonatorum a. Haemophilus influenza 246. What information should 42 year old patient with newly diagnosed diabetes receive about exercise a. Snack before exercise 247. Which of the following are characteristic of patients with type 2 diabetes? 1 beta cell destruction 2 high body mass 3 central obesity 4 unexplained weight loss a. 2, 3 248. The NP correctly diagnoses iron deficiency anemia in a female patient whose lab report reveals a. An increased total iron binding capacity249. A 19 year old female reports that she is having panic attacks. The NP knows that panic attacks are characterized by a. Sudden onset of intense fear or terror 250. At what age is screening most helpful in detecting scoliosis a. 12 to 14 251. A 66 year old female presents to your clinic. She states that yesterday evening she had chest pain for 20-30 minutes. Which finding most strongly correlates with myocardial infarction a. Elevated troponin I levels 252. A 27 year old female patient with epilepsy is well controlled with phenytoin. She requests information about contraception. The NP should instruct her that a. The effectiveness of an oral contraceptive may be reduced 253. Which of the following statements concerning informed consent is false a. It is not legally binding if the patient cannot write his or her name 254. With a history of ankle sprain, the NP would most likely elicit a history of a. Ankle inversion 255. The diagnosis of Meniere’s disease is based on a. Exclusion of other pathologies 256. A patient presents with an inflamed upper eyelid margin. The conjunctiva is red and there is particulate matter along the upper eyelid. The patient complains of a sensation that there is something in my eye. What is the diagnosis and how should it be treated a. Blepharitis, treat with warm compresses and gentle debridement with a cotton swab 257. A 43 year old female patient complains of dull ache around both ankles after a days work as a cashier. Her symptoms are relieved by sitting and elevating her legs. She reports ankle edema at the end of the day. What is the most likely cause of these problems A. Varicose veins 258. An elder’s caretaker is planning the elders menus. The elder is on a bland diet. The NP reviews the meal plans and notes that which of the following is not appropriate a. Coffee, tea, carbonated sodas, orange juice259. Microscopic examination of expressed prostatic secretions and post prostate massage urine is positive for WBCs >10 to 20/HPF and negative for bacteria. The most likely diagnosis is a. Chronic nonbacterial prostatisis 260. A 41 year old woman presents with multiple painful vesicular lesions in the vulvar area. She has been with the same sexual partner for 25 years. Which of the following is the most likely diagnosis a. Herpes simplex 261. In the differential diagnosis of thyroid dysfunction, the typical patient with hypothyroidism has all of the following characteristic features except a. Nervousness and tremor 262. A 16 year old male presents with mid sore throat, fever, fatigue, posterior cervical adenopathy, and palatine petechiae. Without a definitive diagnosis for this patient what drug would be the least appropriate to prescribe a. Ampicillin 263. Which of the following is not a component of the fetal biophysical profile a. Gestational age estimate 264. Which of the following ecchymotic lesions in a 4 year old patient most likely represents a possible bleeding disorder a. 7 cm on the abdomen 265. Today, a patient is diagnosed with iron deficiency anemia and started on a daily iron supplement. In order to best assess the adequacy of supplementation, the np would appropriately order a follow up a. Hemoglobin level in 1 month 266. Antibiotic administration has been demonstrated to be of little benefit in the treatment of which of the following disease processes a. Acute bronchitis 267. A 44 year old male Caucasian patients blood pressures are consistently 170 and 100-110 diastolic. He has no significant medical history and is of normal weight. His lab tests and ecg are normal. What is the appropriate action at this time a. Prescribe an ACE inhibitor and a low dose thiazide diuretic268. A 50 year old male is scheduled for his annual wellness visit. The NP would appropriately recommend all of the following except a. Monthly self testicular examination and annual clinical testicular examination 269. A 2 week old infant is brought to the NP clinic after a difficult delivery. The mother says the infant fusses when handled or picked up. On physical examination the NP notes decreased movement of the right arm during the moro reflex and crepitus on palpation of the right clavicle. The diagnosis is fracture of the clavicle. The recommended management is A. Instructions to the parents to handle the neonate gently 270. A 3 year old female with a palpable right upper quadrant abdominal mass, anemia, and fever is being evaluated for Wilm’s tumor. Which of the following diagnostic tests would be most useful a. Abdominal ultrasound 271. A patient presents with pruritic lesions on both knees. There are visible silver scales. How should his condition be managed a. Topical corticosteroid cream 272. Persons with thalassemia should avoid a. Iron replacement therapy 273. Which of the following is the most important diagnosis to rule out in the adult patient with acute bronchitis a. Pneumonia 274. A patient on oral contraceptives complains of breast fullness, tenderness and some nausea. The patient is concerned. How should the np manage this a. Chance the pill to one containing less estrogen 275. The client with iron deficiency anemia should be advised to take the iron supplement a. On an empty stomach between meals 276. A 45 year old diabetic patient has periorbital cellulitis secondary to sinus infection. What course of action should the np take a. Consider collaboration with a physician regarding antibiotic treatment, culture and subs277. A 26 year old male patient in good health has a blood pressure of 145/80. How soon should his blood pressure be reassessed a. 1 month 278. An 8 year old is sent home from school with a mucopurulent discharge from his eye. He is brought to the nurse practitioner for treatment. What is the most appropriate intervention a. Polysporin 279. Which of the following tests are used for diagnosis of cystic fibrosis a. Sweat test 280. Swan neck and boutonniere deformities are typical clinical findings in the later presentation of a. Rheumatoid arthritis 281. The minimum age for routine immunization with the MMR vaccine is a. 6 months 282. A 32 year old woman reports a 10 year history of stuffy nose with clear rhinorrhea, sneezing, and itchy watery eyes. The MOST effective class of medications for this condition is a. Leukotriene receptor antagonists 283. A 65 year old man complains of shoulder pain lasting 3 months. He denies trauma and states that initially the pain was severe. However, over time the pain has lessened and stiffness of the joint has increased. Which diagnosis should be included as a differential a. Adhesive capsulitis 284. A 50 year old homeless woman presents with persistent, productive cough for the past month, as well as night sweats and weight loss. The most appropriate approach to diagnosis of this patient is a. Mantoux test 285. A 35 year old black man presents with a complaint of superficial irritation of the skin after shaving his facial hair. The physical examination identifies small raisederythematous lesions with pruritic pustules at the site. This is an assessment finding consistent with a. Folliculitis 286. A 20 year old woman with depression states that she is thinking about killing herself. The next step would be to ask her a. Do you have a plan for killing yourself 287. The term used to describe breath sounds audible over the center part of the anterior chest is a. Bronchovesicular 288. A 32 year old man presents with a 24 hour history of fever, diarrhea, nausea, and dehydration. The symptom most suggestive of infection with an invasive organism is a. Fever 289. Treatment for which of the following conditions can result in the development of cataracts a. Asthma 290. When evaluating a 55 year old man for his understanding of self testicular exams, he needs further instruction if he says that a. I should make sure to perform self examination every 2-3 months to assess for hard nodules 291. The recognition and validation of knowledge, skills, competency and qualifications to function as an advanced practice registered nurse is a. Certification 292. X-link dominant disorders a. Affects only men 293. Which of the following diagnoses would not be considered a possible complication in a 61 year old man with prostatitis a. Erectile dysfunction 294. A 26 year old woman who is 10 weeks pregnant has a negative rubella titer. This patient should a. Wait until after delivery to be vaccinated295. A 62 year old woman is diagnosed with shingles and concerned about spreading the virus to others. Which of the following statements is correct a. The shingles virus can also be spread in the prodrome period, prior to the development of the maculopapular rash and vesicle eruption 296. A 10 year old boy is diagnosed with otitis externa. The symptom not usually seen in patients with otitis externa is a. Fever 297. The daily performance of cardiovascular exercise has many benefits, including a. Increased high density lipoprotein levels 298. The nurse practitioner documents a blowing, high pitched decrescendo murmur at the left sternal border. This finding is consistent with a. Aortic regurgitation 299. A 20 year old veterinarian intern presents with right inguinal lymphadenopathy. The nurse practitioner suspects a typical case of cat scratch disease (CSD) and knows that a. Antibiotics are not recommended and the disease is self limiting 300. Depressed mood and teeth erosion are present in a 16 year old girl. Initially she should be referred to a a. Cognitive behavioral therapist 301. Which of the following is a violation of HIPPA a. Releasing sensitive information about a teenage patient to a parent without consent 302. A 49 year old man presents with an erythematous swollen and painful proximal interphalangeal joint on the right foot. Which medication should not be initiated during an acute attack a. Allopurinol 303. A patient presents with a foreign body sensation in the eye, photophobia and tearing after removal of a contact lens. A fluorescein stain confirms a corneal abrasion. The best choice for treatment is a. Oral anti-inflammatory medication and use of an eye patch 304. Which answer best describes hidradenitis suppurativa a. It is an infection of the sweat gland305. When a payment arrangement is establish for service and a set fee per patient per month is paid for the provision of primary care services, it is known as a. Fee for service 306. A 59 year old nurse has varicose veins in her lower extremities. Which of the following can be attributed to varicose veins a. Bilateral orthostatic edema 307. Which of the following diagnostic tests has the highest sensitivity and specificity in diagnosing deep vein thrombosis (DVT) a. Compressive duplex ultrasound 308. Ophthalmoscopic examination of the retina reveals arteriovenous nicking (AV nicking). This appears as if the a. Artery crosses the vein and causes the vein the bulge, forming an hourglass shape 309. On assessment, a patient is diagnosed with koilonychia. This is indicative of a. Iron deficiency anemia 310. The history and assessment of a 50 year old woman identify stress incontinence. The patient asks what could cause this condition. The most appropriate response is a. Constipation 311. A 70 year old man has been diagnosed with tinnitus. The initial workup should include a. Examination of the ear canals for cerumen 312. Mr. Briar has had type 2 diabetes for 6 years. He is now diagnosed with elevated low density lipoprotein (LDL) cholesterol. The best pharmacologic management to manage his lipids is a. A statin 313. A 38 year old woman complains of a headache. A finding that could indicate a serious underlying cause is a. A headache lasting longer than 24 hours 314. A 60 year old man has been diagnosed with heart failure. Which item would be most beneficial in managing his condition at home a. A bathroom scale315. An 18 year old woman with anorexia nervosa returns for a 1 month evaluation of the effectiveness of cognitive behavioral therapy (CBT). On examination, she is hypotensive, has a heart rate of 55, and a body temp 96F. Her weight has fallen below the 75th percentile. The nurse practitioner should a. Order a selective serotonin reuptake inhibitor (SSRI) 316. A 5 year old boy is brought in with lesions around the mouth and nose. If the presentation is impetigo, the nurse practitioner would expect to find a. Bullae and crusting of the lesions 317. A 30 year old mother of three school aged boys complains of a sore throat. The least worrisome finding would be a a. Nontender supraclavicular node 318. A 44 year old man reports gradual hearing loss over the past 3 months. Results of the Rinne tuning fork test are right ear air conduction (AC) greater than bone conduction (BC) and left ear BC greater than AC. This finding indicates a. Conductive hearing loss in the right ear 319. A 16 year old patient is advised to take nonsteroidal anti-inflammatory drugs (NSAIDs) for dysmenorrhea because they a. Inhibit prostaglandin synthesis 320. When examining the breathing pattern of a 20 year old man, the diaphragm moves upward during inspiration and downward during expiration. The type of breathing is documented as a. Thoracoabdominal paradox 321. Ms. Whitt, a mother of five children, is complaining of stress urinary incontinence. The next step in evaluating this condition is to recommend a. A voiding diary 322. The characteristic that is associated with the highest risk for cervical cancer in a black woman in her 30s is a a. Human papillomavirus (HPV) infection 323. A risk factor associated with hepatitis a infection in a young woman is a. Eating raw oysters324. According to the national heart lung and blood institute (NHLBI), a normally healthy 12 year old should undergo lipid screening a. Now 325. Cervical cancer screening guidelines recommend that screening begin at age a. 21 years 326. A 27 year old man presents with a blood pressure of 150/95 mmHg. Gynecomastia and shrunken testicles are noted on exam. The nurse practitioner suspects a. Anabolic steroid use 327. An 81 year old woman comes to the office with mental status changes. Her daughter says that she has been confused for the past 3 days, and that she has complained of fatigue and headache. The initial diagnostic test in the evaluation of this patient is a a. Urinalysis 328. How long after delivery should a pregnant patient with hypothyroidism have a TSH drawn a. 1 week 329. A 25 year old pregnant woman is diagnosed with a urinary tract infection. The safest antibiotic to treat this patient is a. Nitrofurantoin 330. A 6 year old boy exhibits nuchal rigidity. In children, this finding is indicative of a. Meningeal irritation 331. A 36 year old man presents with facial plethora, wide reddened striae, and easy bruising. These findings are most consistent with a. Cushing’s syndrome 332. When performing a physical examination in a patient with suspected acute otitis media, a positive tug test will elicit pain when a. The auricle is gently pulled downward 333. Mr. Wilson is started on a selective serotonin reuptake inhibitor (SSRI) for depression. Which one of the following statements is true about SSRIs a. Many over the counter drugs can interact with an SSRI 334. A 13 year old girl has never received the varicella immunization and her history is unclear about varicella infection. The initial step is toa. Administer two doses within 4 weeks of each other 335. A 10 month old infant girl has a history of prematurity and prolonged febrile seizures occurring on two occasions. When discussing the prognosis with the mother, the nurse practitioner informs her that a. Febrile seizures are usually benign and alone do not signify epilepsy 336. An extract of the fruit of saw palmetto is sold over the counter and is commonly used to a. Improve urine flow in older men 337. A 55 year old woman with diabetes complains of eye pain and headaches and states that she sees halos around bright lights. The differential diagnosis would include a. Glaucoma 338. A 40 year old man has suspected epididymitis. Which test is best to assess the probability of this condition? a. Prehn’s sign 339. A 42 year old man presents with dyspnea of sudden onset. This complaint could indicate a. Acute pulmonary embolism 340. A 48 year old man presents with a fourth acute attack of gout in 6 months. To prevent future acute flare ups, the best choice of medication is a. Probenecid (Benemid) 250 mg PO BID 341. Bladder catheterization for postvoid residual urine volume and a voiding diary are beneficial in identifying a. Urge incontinence 342. Symptoms of fatigue, occasional shortness of breath, and paroxysmal nocturnal dyspnea in a 58 year old woman could be indicators of a. Microvascular coronary dysfunction 343. Symptoms of trichomoniasis in a 35 year old man could include a. Painful and swollen testicles, painful bowel movements 344. Approximately 1 hour ago, a 10 month old infant experienced a febrile seizure. The mother should be informed thata. Ibuprofen (Advil) and acetaminophen (Tylenol)should be administered every 2 hours for fever management 345. Which of the following diagnoses is most likely to be considered as a differential diagnosis in the presence of anuria? a. Chronic kidney disease 346. An infant presents for a routine exam. Anticipatory guidance about teething should be discussed with the parent when the infant is a. 4 months old 347. The NP treats a family of four with albendazole (Albenza) for pinworms. The mother inquires about reinfection and the NP explains that a. A second dose might be indicated in 2 weeks if symptoms persist 348. A 19 year old patient with a negative rapid strep test has a throat swab that is positive for beta hemolytic streptococcus. What is the best action by the NP a. Prescribe an antibiotic 349. When educating a patient about the initiation of insulin, the NP should explain that a common effect of insulin use is a. Injection site injection 350. At what age should anticipatory guidance about tobacco, alcohol, and drugs take place a. 11-12 years old 351. The diagnostic criteria for chronic kidney disease do not include a. Identification of the underlying cause 352. A 40 year old man presents with persistent wheezing localized to one area of the lung. It is most noticeable at night, when he experiences paroxysmal coughing. The most likely diagnosis is a. Bronchial neoplasm 353. The term used to describe a difference of 10 mm Hg between the highest systolic pressure reading and the lowest systolic pressure is a. Paradoxical pulse 354. A 44 year old man with new onset gout should be advised to lose weight and adjust diet bya. Limiting intake of animal protein, increasing intake of plant based proteins 355. A 67 year old patient with a history of diabetes has a total cholesterol level of 195mg/dl. LDL level of 129 mg/dl, HDL level of 35 mg/dl, and triglyceride of 100 mg/dl. The most appropriate treatment is a. Pravastatin (Pravachol) 356. A 16 year old baseball pitcher presents with symptoms of medial epicondylitis. On assessment, the NP should suspect a. Pain with passive wrist flexion and a weak grip 357. A 17 year old girl has been taking an over the counter decongestant for weeks without adequate relief from persistent nasal congestion, hoarseness, and occasional nose bleeds. On examination, boggy mucous membranes and a small perforation in the nasal septum are observed. These findings are most consistent with a. Severe allergic rhinitis 358. The most appropriate initial treatment of idiopathic facial nerve palsy (Bell’s palsy) for a patient who began to notice a facial droop 2 days prior is a. Oral glucocorticoids 359. The NP is teaching a patient who is newly diagnosed with diabetes how to properly administer insulin glargine (Lantus). She explains that he can expect the medication to start working within 1 hour and that it has a duration of approx. a. 24 hours 360. Which of the following symptoms would be most suggestive of chlamydia trachomatis infection in a woman a. Thick, cloudy or bloody vaginal discharge with anal itching 361. Mild bladder prolapse in a 42 year old woman may be treated with a. Vaginal topical estrogen cream 362. When determining whether an infant is on track in terms of gross motor skill development, it is important to know that he can roll from prone to supine by a. 4 months of age 363. A patient can be diagnosed with pernicious anemia if a. Her mcv is <80364. During routine follow up of a patient with obstructive sleep apnea, the NP knows that the patient may need to be referred for adjustment of positive airway pressure therapy if he experiences a. Headache upon awakening 365. A breastfeeding mother presents with symptoms of mastitis. Which of the following is not included in the clinical approach to treatment a. Cessation of lactation 366. When treating a woman with urinary incontinence, the lifestyle medication that is not appropriate is a. Reducing fluid intake to less than 1 liter per day 367. Which credential is typically listed first behind a NP’s name a. The highest academic degree earned 368. Which one of the following is not part of the diagnostic criteria for attention deficit disorder in the diagnostic and statistical manual of mental disorders a. Symptoms present at home and at school 369. A 2 year old boy presents with shortness of breath temp of 103Fand lethargy. Four weeks ago, he underwent surgery to repair a congenital heart defect. Based on his past history and these symptoms, the next step should be to a. Refer him to the ED for direct pediatric admission 370. After completion of a 7 day course of doxycycline for the treatment if chlamydia, an 18 year old adolescent returns to the clinic for a follow up exam. Symptoms appear to have resolved and the NP instructs the patient to return for retesting a. 2 weeks after completion of therapy 371. The most common cause of bacterial pneumonia in 2 year old is a. S. pneumoniae 372. The most likely history or physical finding in a patient with migraine headaches is a. Focal deficits 373. The best method for preventing gonorrhea and chlamydia in teenagers is to a. Encourage abstinence or the use of condoms 374. An example of secondary prevention is to a. Obtain an annual mammogram after age 40 years375. Which immunization is contraindicated during pregnancy a. Tdap 376. A 6 year old girl was treated for pinworms 1 week ago and now presents with scattered honey-colored crusts of nonbullous lesions on her right hand. The NP should a. Prescribe topical mupirocin BID for 5 days 377. A 20 year old woman states that her 45 year old mother was recently diagnosed with breast cancer. The best tool to assess her risk for developing breast cancer would be the a. Claus assessment model 378. White or grayish/brown speckling on the periphery of the iris of a 6 month old boy with down syndrome is documented as a. Brushfield spots 379. A 16 year old boy is diagnosed with genital warts and wonders what caused this condition. The appropriate response would be that the most common cause is a. HPV 380. What type of seizure is characterized by 2-15 seconds of staring, eye flutters, and automatisms a. Petit mal seizure 381. Laboratory findings for a 66 year old woman show a low serum iron level, elevated total iron binding capacity, and low serum ferritin level. These results are consistent with a. Iron deficiency anemia 382. The diagnostic criterion for COPD is FEV1/FVC ratio of a. Less than 70% 383. The risk of developing metabolic alkalosis in a 48 year old man with a 35 pack year smoking history is most likely related to a. Diarrhea 384. Which condition is not likely to have an associated symptom of hemoptysis a. Bronchiectasis 385. An ankle brachial index (ABI) to determine if a patient has peripheral arterial diseasea. Is positive for the disease if the ABI reading is <0.9 386. An elevated alpha fetoprotein (AFP) level is documented in a 34 year old pregnant woman. The most likely reason for the elevation is that a. She has an undiagnosed miscarriage 387. A 45 year old man places his hand over his sternum and complains of nausea and chest pain. This gesture could be suggestive of a. Angina pectoris 388. A 40 year old woman who underwent gastric bypass surgery approximately 12 weeks ago returns for follow up. Labs reveal a hematocrit of 33%, hemoglobin of 11, MCV of 72, and mch of 26. Based on these results, the best action for the NP to initiate next is to a. Initiate oral iron supplementation 389. The symptom not associated with an anterior cystocele in a 40 year old woman is a. Lower back pain 390. The most common cause of urethritis in men is a. Neisseria gonorrhoeae 391. When using the gtpal system to document a woman’s maternal history, the “t” stands for term births, which is considered delivery after a. 37 weeks 392. A 62 year old patient is complaining of pain and stiffness in both hands. During the examination, the NP notes hard, bony swelling in the distal interphalangeal joints. These are known as a. Heberden nodes 393. Persistent facial redness, flushing, dryness, and sensitivity of the cheeks that worsens with exercise and with ingestion of spicy foods are the presenting symptoms of a 29 year old woman. These symptoms are most likely associated with a. Rosacea 394. A 45 year old woman receives immunotherapy injections weekly, along with fexofenadine and fluticasone nasal spray daily. Today she is diagnosed with a viral upper respiratory infection. The most appropriate intervention would be to a. Add an oral decongestant395. Chronic bacterial prostatitis is not likely to be caused by a. Eubacterium species 396. The most common congenital heart defect is a. Ventricular septal defect 397. A 6 month old infant boy with a history of ventricular septal defect presents with a poor appetite wheezing cough and shortness of breath. To evaluate these symptoms the NP should order a. An echocardiogram 398. Expected spirometry readings in a patient who has chronic bronchitis include a. Decreased forced vital capacity 399. A 30 year old woman has experienced unilateral palsy of the right cheek for the past 36 hours. Which cranial nerve is most likely to be involved a. VII 400. A 38 year old man has polycythemia vera and is scheduled for phlebotomy. The goal of therapeutic phlebotomy is to reduce blood viscosity. It is aimed at bringing which component of the cbc into normal range a. Hematocrit 401. A 48 year old woman has a history of diethylstilbestestrol exposure while in utero. Because of her history, this patient may be at increased risk for a. Vaginal and cervical cancer 402. A lack of fat in a child’s diet can negatively impact the growth and development of the brain. Therefore, limiting fat consumption should not occur before at least a. 2 years of age 403. Recommendations intended to optimize patient care and that are informed by a systematic review of evidence and an assessment of the benefits and harms of alternative care options are known as a. Clinical practice guidelines 404. A 56 year old woman states that she is experiencing shock like intermittent right sided pain in her lips, face, mouth and cheek when yawning. It subsides abruptly. The diagnosis is most likely a. Trigeminal neuralgia405. When initiating a thiazolidinedione for the treatment of type 2 diabetes in an older adult the best practice would be to a. Check liver function 406. A patient with mechanical heart valve is receiving warfarin. Once the inr is stabilized at therapeutic levels it should be monitored at least a. Monthly 407. A straightleg raise in the supine position is positive on the left side between 30 and 60 degrees. This most likely indicates a. Lumbar disc herniation 408. Complications of untreated chlamydia in a 40 year old woman are not likely to include a. Pyelonephritis 409. The NP should expect adolescents between the ages of 12 and 14 to a. Develop a stronger sense of right and wrong 410. Acetaminophen 500 mg tid is prescribed for an 85 year old woman to treat left knee pain caused by osteoarthritis. Acetaminophen has not been effective in relieving her pain, therefore, the next step should be a. Order physical therapy 411. A patient with type 2 diabetes is started on insulin glargine. To reduce the risk of hypoglycemia, which drug class should be discontinued a. Sulfonylurea 412. A patient with acute epicondylitis needs further education about the management of symptoms if he says a. I should apply ice with pressure directly to the skin of the epicondyle for 20 mins 3 times a day 413. Which one of the following is a diagnostic criterion for bacterial vaginosis a. Clue cells 414. A 42 year old woman with a history of surgically induced menopause is complaining of stress incontinence. She is found to have mild bladder prolapse. The most appropriate treatment for this patient is a. Vaginal topical estrogen cream415. When considering preventive screening in adults, which of the following statements is incorrect a. Risks associated with preventive screening may include overdiagnosis of conditions that may never become clinically apparent 416. A high potency steroid should be avoided on the face and which other area on the body a. The back of the hands 417. A young woman presents with a swollen painful ankle after stepping in a hole. History reveals that she has likely inverted her ankle, which is likely to cause a a. Ligamentous injury of the lateral malleolus 418. The leading cause of vision loss among patients older than 65 years is a. Age related macular degeneration 419. Which of the following is true about multiphasic forms of oral contraceptives a. They contain variable doses of one or both hormones during active pill phase 420. Aa 20 year old pitcher is diagnosed with epicondylitis of the elbow. The assessment is most likely to reveal a. Pain that radiates into the wrist 421. A 30 year old man complains of heaviness in his scrotum and palpable mass of the inguinal canal. The least likely differential diagnosis is a. Inguinal hernia 422. A NP student is preparing to take the certification exam for practice as a NP. Which of the following is a certifying body for NPs a. AANP 423. When prescribing pregabalin a schedule V substance for a patient with neuropathy the prescription should not a. Be faxed 424. A patient presents with loss of function and sensation in the fifth digit. The NP identifies hypothenar atrophy which is suggestive of a. An ulnar nerve disorder 425. A 13 year old boy presents with an abrupt onset of high fever and sore throat and he exhibits the sniffing position. These symptoms are consistent witha. Epiglottitis 426. After providing instructions for activities of daily living to a 66 year old man with heart failure the NP understands that he needs further education after he states a. I should go to the hospital if my morning blood pressure is higher than 130/80 427. A 68 year old patient presents with a painful erythematous maculopapular rash over the left posterior scapular area. The rash does not extend beyond midline. The clinician suspects a. Herpes zoster 428. A 40 year old woman with a longstanding history of asthma has been treated with inhaled steroids for about 20 years. Assessment of this patient might reveal a. Dysphonia 429. A 68 year old man who underwent a mitral valve replacement 3 months ago has been on warfarin therapy. This patient needs to be informed that the warfarin a. Will need to be stopped 4 days prior to dental surgery and resumed immediately afterward 430. A 40 year old complains of breathing difficulty that awakens him from sleep. He states that when he sits up his breathing improves. The most likely diagnosis is a. Paroxysmal nocturnal dyspnea b. Asthma c. COPD d. Emphysema 431. A sleep disturbance that might be reported by a 20 year old woman diagnosed with anxiety is a. Early morning wakening b. Excessive drowsiness c. Difficulty falling and/or staying asleep d. Never feeling tired 432. Medicare Part A coverage provides payment for a. Inpatient/hospital services b. Outpatient/clinic services c. Physical therapyd. Prescription drugs 433. A 78 year old woman is diagnosed with pneumonia. Further examination reveals that she is moderately dehydrated. However, her chest x-ray does not demonstrate infiltrates because a. Infiltrates are often absent when a patient is dehydrated 434. A 30 year old premenopausal woman presents with a breast lump. The lump is nontender and is approximately 1 cm in diameter. The NP should a. Order a screening mammogram b. Order a diagnostic mammogram c. Order an ultrasound d. Have her return 3-10 days after her next menses 435. An adult man complains of two episodes of erectile dysfunction. The LEAST important assessment of sexual dysfunction is a. Erectile reserve b. Age c. Interpersonal conflict d. Partner interview 436. The assessment of a 72 year old man causes the NP to have concerns that include risk for suicide. Which one of the following is not a risk factor for suicide in older adults? a. Chronic pain b. comorbid physical illness c. family history of suicide d. retirement 437. A 25 year old woman complains that the presence of hirsutism over her face and chest is negatively affecting her quality of life. The most effective initial pharmacologic therapy for this patient would be a. 5 alpha reductase inhibitors b. Progestin only contraceptive c. Estrogen progestin contraceptive d. Biguanides438. A lack of fat in a child’s diet can negatively impact the growth and development of the brain. Therefore, limiting fat consumption should not occur before at least a. 2 years of age 439. A 30 year old patient is starting a career in the healthcare industry. The NP knows that the hepatitis B vaccine a. Can be given at any age 440. A 70 year old woman has diffuse lentigo spots on her arms that have developed over time. The NP should inform the patient that a. The lesions must be removed and sent for pathology testing b. These markings of the skin are treated with topical fluorouracil cream c. These spots are harmless and do not require treatment or further testing d. Liver enzymes should be obtained to assess for underlying disease 441. A NP is treating a 12 year old child for suspicious injuries. Which statement is correct about reporting suspected child abuse in the US? a. Only social workers and physicians are considered mandatory reporters b. A healthcare provider must be certain prior to reporting suspected abuse c. The duty to report supersedes patient confidentiality d. Each state varies according to obligation to report suspected abuse 442. A 36 year old woman’s pap results reveal precancerous cells. The NP knows that the most common type of cervical cancer is a. Basal cell carcinoma b. Squamous cell carcinoma c. Ductal cell carcinoma d. Adenocarcinoma 443. The vaccine that can minimize complications related to the disease that causes painful localized, blistering rash that follows a dermatomal pattern is the a. Small pox vaccine b. varicella vaccine c. herpes zoster vaccine d. meningococcal vaccine444. A 45 year old man complains of acute flank pain radiating toward the groin and testicles. For suspected nephrolithiasis, the first line treatment regimen includes Toradol IM plus a. Cipro and hydrocodone b. Hyoscyamine and Bactrim orally c. Indocin and Macrobid d. Indocin and Flomax 445. A sexual maturity assessment of a 12 year old boy indicates that he is starting puberty. The most likely finding was a. Testicular enlargement b. Presence of pubic hair c. Rapid increase in vertical growth d. Lengthening of the penis 446. A mother is inquiring about the HPV vaccine for her son. The NP advises her that this vaccine is recommended a. At 11 to 12 years of age b. Between 13 and 16 years old c. Between 13 and 26 years old d. Only for boys and young men who are sexually active at the time of vaccination 447. Evaluation of cognitive function in a 70 year old man reveals that he has transient difficulty recalling names of people, objects or certain details of specific events. Reasoning and judgment skills are intact. These symptoms are consistent with a. Tia b. Dementia c. Benign forgetfulness d. Alzheimers 448. A patient has been taking a histamine 2 blocker for reflux symptoms. Which H2 blocker is associated with the most drug-drug interactions? a. Pepcid b. Tagamet c. Zantacd. Actid 449. A 51 year old man with atrial fibrillation is taking warfarin. Therapeutic anticoagulation should be maintained with an a. INR 2-3 450. An adult patient is diagnosed with acute bacterial rhinosinusitis. He was treated with azithromycin 4 weeks ago for sore throat. He is allergic to penicillin. The next best treatment is a. Augmentin b. Rocephin c. Doxycycline d. Zinacef 451. A NP-led clinic has received a wood’s lamp. The NP knows that this device is NOT beneficial in diagnosing a. Tinea corporis b. Corneal abrasion c. Tinea capitis d. Atopic dermatitis 452. When evaluating a patient with suspected depression, the finding that is required to diagnose depression is a. Altered sleep patterns b. Weight gain c. Anhedonia d. Feeling of worthlessness 453. A 33 year old woman diagnosed with depression has been treated for 8 months with Lexapro 20 mg daily. She states that her mood is better and she would like to discontinue the medication. The NP should reassess her symptoms and if improved: a. Discontinue the medication now b. Discontinue the medication in a week c. Decrease the medication by 10 mg then stop it a week later d. Decrease the dosage to 10 mg today and reassess symptoms in 2 weeks454. A 52 year old patient with type 2 diabetes has been receiving Levaquin for the treatment of acute sinusitis. What new symptom indicates that this patient will need IV therapy? a. Frontal headache b. Tooth pain c. Periorbital swelling d. Copious nasal discharge [Show More]

Last updated: 9 months ago

Preview 1 out of 70 pages

Reviews( 2 )

user-profile-pic


by Kate O'Brien · 6 months ago

read it

Thank you for the review. by Bobweiss. 6 months ago

user-profile-pic


by Bobweiss · 6 months ago

Thank you for the review.

$16.50

Add to cart

Instant download

Can't find what you want? Try our AI powered Search

OR

GET ASSIGNMENT HELP
278
6

Document information


Connected school, study & course


About the document


Uploaded On

Mar 18, 2021

Number of pages

70

Written in

Seller


seller-icon
Bobweiss

Member since 3 years

39 Documents Sold


Additional information

This document has been written for:

Uploaded

Mar 18, 2021

Downloads

 6

Views

 278

Recommended For You


$16.50
What is Browsegrades

In Browsegrades, a student can earn by offering help to other student. Students can help other students with materials by upploading their notes and earn money.

We are here to help

We're available through e-mail, Twitter, Facebook, and live chat.
 FAQ
 Questions? Leave a message!

Follow us on
 Twitter

Copyright © Browsegrades · High quality services·